A problem in real analysis of a topological nature












3














Let $f: R to R$ be a function such that the closure of its graph contains as a subset the graph of a uniformly continuous function. Does there exist a dense subset $S$ of $R$ such that the restricted function $f|S: S to R$ is uniformly continuous?










share|cite|improve this question


















  • 1




    If $f|_S$ is uniformly continuous then it can be natuarlly extended to a uniformly continuous function on the closure of $S$. Hence $f$ itself has to be uniformly continuous. If you weaken the assumption to "there is a sequence of sets $S_n$ such that their union is dense in $R$ and f restricted to $S_n$ is uniformly continuous on $S_n$" then it's true for measurable functions by Lusin's Theorem.
    – Martin Kell
    6 hours ago






  • 1




    @MartinKell: It's true that $f|_S$ will extend to a uniformly continuous function (call it $g$), but $f$ need not agree with $g$ on $S^c$, so we cannot conclude that $f$ is uniformly continuous.
    – Nate Eldredge
    6 hours ago










  • @Nate: true, the conclusion was too quick. However it shows that a dense subset where f is uniformly continuous is rather restrictive. A counterexample is just sin(1/x) with arbitrary value at zero: 1st for all but the zero value the function is continuous so it’s graph closed at (x,f(x)), 2nd for no value at 0 is the function continuous. Especially it’s not uniformly continuous. Btw. usng non-compactness is $mathbb{R}$ one may contruct even continuous counterexamples.
    – Martin Kell
    6 hours ago
















3














Let $f: R to R$ be a function such that the closure of its graph contains as a subset the graph of a uniformly continuous function. Does there exist a dense subset $S$ of $R$ such that the restricted function $f|S: S to R$ is uniformly continuous?










share|cite|improve this question


















  • 1




    If $f|_S$ is uniformly continuous then it can be natuarlly extended to a uniformly continuous function on the closure of $S$. Hence $f$ itself has to be uniformly continuous. If you weaken the assumption to "there is a sequence of sets $S_n$ such that their union is dense in $R$ and f restricted to $S_n$ is uniformly continuous on $S_n$" then it's true for measurable functions by Lusin's Theorem.
    – Martin Kell
    6 hours ago






  • 1




    @MartinKell: It's true that $f|_S$ will extend to a uniformly continuous function (call it $g$), but $f$ need not agree with $g$ on $S^c$, so we cannot conclude that $f$ is uniformly continuous.
    – Nate Eldredge
    6 hours ago










  • @Nate: true, the conclusion was too quick. However it shows that a dense subset where f is uniformly continuous is rather restrictive. A counterexample is just sin(1/x) with arbitrary value at zero: 1st for all but the zero value the function is continuous so it’s graph closed at (x,f(x)), 2nd for no value at 0 is the function continuous. Especially it’s not uniformly continuous. Btw. usng non-compactness is $mathbb{R}$ one may contruct even continuous counterexamples.
    – Martin Kell
    6 hours ago














3












3








3







Let $f: R to R$ be a function such that the closure of its graph contains as a subset the graph of a uniformly continuous function. Does there exist a dense subset $S$ of $R$ such that the restricted function $f|S: S to R$ is uniformly continuous?










share|cite|improve this question













Let $f: R to R$ be a function such that the closure of its graph contains as a subset the graph of a uniformly continuous function. Does there exist a dense subset $S$ of $R$ such that the restricted function $f|S: S to R$ is uniformly continuous?







real-analysis






share|cite|improve this question













share|cite|improve this question











share|cite|improve this question




share|cite|improve this question










asked 8 hours ago









James Baxter

1668




1668








  • 1




    If $f|_S$ is uniformly continuous then it can be natuarlly extended to a uniformly continuous function on the closure of $S$. Hence $f$ itself has to be uniformly continuous. If you weaken the assumption to "there is a sequence of sets $S_n$ such that their union is dense in $R$ and f restricted to $S_n$ is uniformly continuous on $S_n$" then it's true for measurable functions by Lusin's Theorem.
    – Martin Kell
    6 hours ago






  • 1




    @MartinKell: It's true that $f|_S$ will extend to a uniformly continuous function (call it $g$), but $f$ need not agree with $g$ on $S^c$, so we cannot conclude that $f$ is uniformly continuous.
    – Nate Eldredge
    6 hours ago










  • @Nate: true, the conclusion was too quick. However it shows that a dense subset where f is uniformly continuous is rather restrictive. A counterexample is just sin(1/x) with arbitrary value at zero: 1st for all but the zero value the function is continuous so it’s graph closed at (x,f(x)), 2nd for no value at 0 is the function continuous. Especially it’s not uniformly continuous. Btw. usng non-compactness is $mathbb{R}$ one may contruct even continuous counterexamples.
    – Martin Kell
    6 hours ago














  • 1




    If $f|_S$ is uniformly continuous then it can be natuarlly extended to a uniformly continuous function on the closure of $S$. Hence $f$ itself has to be uniformly continuous. If you weaken the assumption to "there is a sequence of sets $S_n$ such that their union is dense in $R$ and f restricted to $S_n$ is uniformly continuous on $S_n$" then it's true for measurable functions by Lusin's Theorem.
    – Martin Kell
    6 hours ago






  • 1




    @MartinKell: It's true that $f|_S$ will extend to a uniformly continuous function (call it $g$), but $f$ need not agree with $g$ on $S^c$, so we cannot conclude that $f$ is uniformly continuous.
    – Nate Eldredge
    6 hours ago










  • @Nate: true, the conclusion was too quick. However it shows that a dense subset where f is uniformly continuous is rather restrictive. A counterexample is just sin(1/x) with arbitrary value at zero: 1st for all but the zero value the function is continuous so it’s graph closed at (x,f(x)), 2nd for no value at 0 is the function continuous. Especially it’s not uniformly continuous. Btw. usng non-compactness is $mathbb{R}$ one may contruct even continuous counterexamples.
    – Martin Kell
    6 hours ago








1




1




If $f|_S$ is uniformly continuous then it can be natuarlly extended to a uniformly continuous function on the closure of $S$. Hence $f$ itself has to be uniformly continuous. If you weaken the assumption to "there is a sequence of sets $S_n$ such that their union is dense in $R$ and f restricted to $S_n$ is uniformly continuous on $S_n$" then it's true for measurable functions by Lusin's Theorem.
– Martin Kell
6 hours ago




If $f|_S$ is uniformly continuous then it can be natuarlly extended to a uniformly continuous function on the closure of $S$. Hence $f$ itself has to be uniformly continuous. If you weaken the assumption to "there is a sequence of sets $S_n$ such that their union is dense in $R$ and f restricted to $S_n$ is uniformly continuous on $S_n$" then it's true for measurable functions by Lusin's Theorem.
– Martin Kell
6 hours ago




1




1




@MartinKell: It's true that $f|_S$ will extend to a uniformly continuous function (call it $g$), but $f$ need not agree with $g$ on $S^c$, so we cannot conclude that $f$ is uniformly continuous.
– Nate Eldredge
6 hours ago




@MartinKell: It's true that $f|_S$ will extend to a uniformly continuous function (call it $g$), but $f$ need not agree with $g$ on $S^c$, so we cannot conclude that $f$ is uniformly continuous.
– Nate Eldredge
6 hours ago












@Nate: true, the conclusion was too quick. However it shows that a dense subset where f is uniformly continuous is rather restrictive. A counterexample is just sin(1/x) with arbitrary value at zero: 1st for all but the zero value the function is continuous so it’s graph closed at (x,f(x)), 2nd for no value at 0 is the function continuous. Especially it’s not uniformly continuous. Btw. usng non-compactness is $mathbb{R}$ one may contruct even continuous counterexamples.
– Martin Kell
6 hours ago




@Nate: true, the conclusion was too quick. However it shows that a dense subset where f is uniformly continuous is rather restrictive. A counterexample is just sin(1/x) with arbitrary value at zero: 1st for all but the zero value the function is continuous so it’s graph closed at (x,f(x)), 2nd for no value at 0 is the function continuous. Especially it’s not uniformly continuous. Btw. usng non-compactness is $mathbb{R}$ one may contruct even continuous counterexamples.
– Martin Kell
6 hours ago










1 Answer
1






active

oldest

votes


















3














Consider the following modification of the Dirichlet "popcorn" function:
$$f(x) = begin{cases} 1/q, & text{$x in mathbb{Q}$, $x=p/q$ in lowest terms} \
-1, & x notin mathbb{Q},, x < 0 \
-2, & x notin mathbb{Q}, , x > 0.end{cases}$$

Since every real number can be approximated by rationals with arbitrarily large denominator, the graph of $f$ contains the $x$-axis, which is the uniformly continuous function $0$.



Let $S subset mathbb{R}$ be dense. If $f|_S$ is uniformly continuous, then it extends to a unique uniformly continuous function $g$ on all of $mathbb{R}$, and we have $f=g$ on $S$.



If $S$ contains a negative irrational number $x$, then $g(x) = f(x) = -1$. Let $y$ be any positive number in $S$. If $y$ is rational, we have $g(y) = f(y) = 0$. Then by the continuity of $g$, there would have to be some $z in S$ with $f(z) = g(z) in (-3/4, -1/4)$ which is impossible. If $y$ is irrational, we get a similar contradiction since $g(y) = f(y) = -2$. So $S$ does not contain any negative irrational. Similarly, $S$ does not contain any positive irrational.



So we must have $S subset mathbb{Q}$. But this is similarly impossible. The rationals in $S$ cannot all have the same denominator (in lowest terms), so let $x_1 = p_1/q_1, x_2 = p_2/q_2 in S$, where $q_1 < q_2$. Then by the continuity of $g$ there must be some $y in S$ with $f(y) = g(y) in (frac{1}{q_1}, frac{1}{q_1+1})$, but $f$ never takes on any such value.






share|cite|improve this answer





















  • Slight simplification of the proof: $S$ cannot contain a single rational number, since $f$ is discontinuous at every rational, hence so is $f|_S$ for any dense $S$. Thus $Ssubseteqmathbb Rsetminusmathbb Q$, but then we have problems around $0$.
    – Wojowu
    5 hours ago












  • @Wojowu: It's not quite that simple, I think. For instance, the function $f=1_mathbb{Q}$ is discontinuous at every rational (and every irrational), but its restriction to $S=mathbb{Q}$ is uniformly continuous. So the fact that $f$ is discontinuous at the rationals doesn't immediately rule out the possibility for $S$ to contain rationals. We would have to work a little harder. I guess the key is that $f$ has a "jump" discontinuity at every rational.
    – Nate Eldredge
    5 hours ago












  • You are right, I was a little too quick there. We can either use jump discontinuity as you mention, or we can observe $(q,f(q))$ is an isolated point of the graph.
    – Wojowu
    5 hours ago











Your Answer





StackExchange.ifUsing("editor", function () {
return StackExchange.using("mathjaxEditing", function () {
StackExchange.MarkdownEditor.creationCallbacks.add(function (editor, postfix) {
StackExchange.mathjaxEditing.prepareWmdForMathJax(editor, postfix, [["$", "$"], ["\\(","\\)"]]);
});
});
}, "mathjax-editing");

StackExchange.ready(function() {
var channelOptions = {
tags: "".split(" "),
id: "504"
};
initTagRenderer("".split(" "), "".split(" "), channelOptions);

StackExchange.using("externalEditor", function() {
// Have to fire editor after snippets, if snippets enabled
if (StackExchange.settings.snippets.snippetsEnabled) {
StackExchange.using("snippets", function() {
createEditor();
});
}
else {
createEditor();
}
});

function createEditor() {
StackExchange.prepareEditor({
heartbeatType: 'answer',
autoActivateHeartbeat: false,
convertImagesToLinks: true,
noModals: true,
showLowRepImageUploadWarning: true,
reputationToPostImages: 10,
bindNavPrevention: true,
postfix: "",
imageUploader: {
brandingHtml: "Powered by u003ca class="icon-imgur-white" href="https://imgur.com/"u003eu003c/au003e",
contentPolicyHtml: "User contributions licensed under u003ca href="https://creativecommons.org/licenses/by-sa/3.0/"u003ecc by-sa 3.0 with attribution requiredu003c/au003e u003ca href="https://stackoverflow.com/legal/content-policy"u003e(content policy)u003c/au003e",
allowUrls: true
},
noCode: true, onDemand: true,
discardSelector: ".discard-answer"
,immediatelyShowMarkdownHelp:true
});


}
});














draft saved

draft discarded


















StackExchange.ready(
function () {
StackExchange.openid.initPostLogin('.new-post-login', 'https%3a%2f%2fmathoverflow.net%2fquestions%2f319525%2fa-problem-in-real-analysis-of-a-topological-nature%23new-answer', 'question_page');
}
);

Post as a guest















Required, but never shown

























1 Answer
1






active

oldest

votes








1 Answer
1






active

oldest

votes









active

oldest

votes






active

oldest

votes









3














Consider the following modification of the Dirichlet "popcorn" function:
$$f(x) = begin{cases} 1/q, & text{$x in mathbb{Q}$, $x=p/q$ in lowest terms} \
-1, & x notin mathbb{Q},, x < 0 \
-2, & x notin mathbb{Q}, , x > 0.end{cases}$$

Since every real number can be approximated by rationals with arbitrarily large denominator, the graph of $f$ contains the $x$-axis, which is the uniformly continuous function $0$.



Let $S subset mathbb{R}$ be dense. If $f|_S$ is uniformly continuous, then it extends to a unique uniformly continuous function $g$ on all of $mathbb{R}$, and we have $f=g$ on $S$.



If $S$ contains a negative irrational number $x$, then $g(x) = f(x) = -1$. Let $y$ be any positive number in $S$. If $y$ is rational, we have $g(y) = f(y) = 0$. Then by the continuity of $g$, there would have to be some $z in S$ with $f(z) = g(z) in (-3/4, -1/4)$ which is impossible. If $y$ is irrational, we get a similar contradiction since $g(y) = f(y) = -2$. So $S$ does not contain any negative irrational. Similarly, $S$ does not contain any positive irrational.



So we must have $S subset mathbb{Q}$. But this is similarly impossible. The rationals in $S$ cannot all have the same denominator (in lowest terms), so let $x_1 = p_1/q_1, x_2 = p_2/q_2 in S$, where $q_1 < q_2$. Then by the continuity of $g$ there must be some $y in S$ with $f(y) = g(y) in (frac{1}{q_1}, frac{1}{q_1+1})$, but $f$ never takes on any such value.






share|cite|improve this answer





















  • Slight simplification of the proof: $S$ cannot contain a single rational number, since $f$ is discontinuous at every rational, hence so is $f|_S$ for any dense $S$. Thus $Ssubseteqmathbb Rsetminusmathbb Q$, but then we have problems around $0$.
    – Wojowu
    5 hours ago












  • @Wojowu: It's not quite that simple, I think. For instance, the function $f=1_mathbb{Q}$ is discontinuous at every rational (and every irrational), but its restriction to $S=mathbb{Q}$ is uniformly continuous. So the fact that $f$ is discontinuous at the rationals doesn't immediately rule out the possibility for $S$ to contain rationals. We would have to work a little harder. I guess the key is that $f$ has a "jump" discontinuity at every rational.
    – Nate Eldredge
    5 hours ago












  • You are right, I was a little too quick there. We can either use jump discontinuity as you mention, or we can observe $(q,f(q))$ is an isolated point of the graph.
    – Wojowu
    5 hours ago
















3














Consider the following modification of the Dirichlet "popcorn" function:
$$f(x) = begin{cases} 1/q, & text{$x in mathbb{Q}$, $x=p/q$ in lowest terms} \
-1, & x notin mathbb{Q},, x < 0 \
-2, & x notin mathbb{Q}, , x > 0.end{cases}$$

Since every real number can be approximated by rationals with arbitrarily large denominator, the graph of $f$ contains the $x$-axis, which is the uniformly continuous function $0$.



Let $S subset mathbb{R}$ be dense. If $f|_S$ is uniformly continuous, then it extends to a unique uniformly continuous function $g$ on all of $mathbb{R}$, and we have $f=g$ on $S$.



If $S$ contains a negative irrational number $x$, then $g(x) = f(x) = -1$. Let $y$ be any positive number in $S$. If $y$ is rational, we have $g(y) = f(y) = 0$. Then by the continuity of $g$, there would have to be some $z in S$ with $f(z) = g(z) in (-3/4, -1/4)$ which is impossible. If $y$ is irrational, we get a similar contradiction since $g(y) = f(y) = -2$. So $S$ does not contain any negative irrational. Similarly, $S$ does not contain any positive irrational.



So we must have $S subset mathbb{Q}$. But this is similarly impossible. The rationals in $S$ cannot all have the same denominator (in lowest terms), so let $x_1 = p_1/q_1, x_2 = p_2/q_2 in S$, where $q_1 < q_2$. Then by the continuity of $g$ there must be some $y in S$ with $f(y) = g(y) in (frac{1}{q_1}, frac{1}{q_1+1})$, but $f$ never takes on any such value.






share|cite|improve this answer





















  • Slight simplification of the proof: $S$ cannot contain a single rational number, since $f$ is discontinuous at every rational, hence so is $f|_S$ for any dense $S$. Thus $Ssubseteqmathbb Rsetminusmathbb Q$, but then we have problems around $0$.
    – Wojowu
    5 hours ago












  • @Wojowu: It's not quite that simple, I think. For instance, the function $f=1_mathbb{Q}$ is discontinuous at every rational (and every irrational), but its restriction to $S=mathbb{Q}$ is uniformly continuous. So the fact that $f$ is discontinuous at the rationals doesn't immediately rule out the possibility for $S$ to contain rationals. We would have to work a little harder. I guess the key is that $f$ has a "jump" discontinuity at every rational.
    – Nate Eldredge
    5 hours ago












  • You are right, I was a little too quick there. We can either use jump discontinuity as you mention, or we can observe $(q,f(q))$ is an isolated point of the graph.
    – Wojowu
    5 hours ago














3












3








3






Consider the following modification of the Dirichlet "popcorn" function:
$$f(x) = begin{cases} 1/q, & text{$x in mathbb{Q}$, $x=p/q$ in lowest terms} \
-1, & x notin mathbb{Q},, x < 0 \
-2, & x notin mathbb{Q}, , x > 0.end{cases}$$

Since every real number can be approximated by rationals with arbitrarily large denominator, the graph of $f$ contains the $x$-axis, which is the uniformly continuous function $0$.



Let $S subset mathbb{R}$ be dense. If $f|_S$ is uniformly continuous, then it extends to a unique uniformly continuous function $g$ on all of $mathbb{R}$, and we have $f=g$ on $S$.



If $S$ contains a negative irrational number $x$, then $g(x) = f(x) = -1$. Let $y$ be any positive number in $S$. If $y$ is rational, we have $g(y) = f(y) = 0$. Then by the continuity of $g$, there would have to be some $z in S$ with $f(z) = g(z) in (-3/4, -1/4)$ which is impossible. If $y$ is irrational, we get a similar contradiction since $g(y) = f(y) = -2$. So $S$ does not contain any negative irrational. Similarly, $S$ does not contain any positive irrational.



So we must have $S subset mathbb{Q}$. But this is similarly impossible. The rationals in $S$ cannot all have the same denominator (in lowest terms), so let $x_1 = p_1/q_1, x_2 = p_2/q_2 in S$, where $q_1 < q_2$. Then by the continuity of $g$ there must be some $y in S$ with $f(y) = g(y) in (frac{1}{q_1}, frac{1}{q_1+1})$, but $f$ never takes on any such value.






share|cite|improve this answer












Consider the following modification of the Dirichlet "popcorn" function:
$$f(x) = begin{cases} 1/q, & text{$x in mathbb{Q}$, $x=p/q$ in lowest terms} \
-1, & x notin mathbb{Q},, x < 0 \
-2, & x notin mathbb{Q}, , x > 0.end{cases}$$

Since every real number can be approximated by rationals with arbitrarily large denominator, the graph of $f$ contains the $x$-axis, which is the uniformly continuous function $0$.



Let $S subset mathbb{R}$ be dense. If $f|_S$ is uniformly continuous, then it extends to a unique uniformly continuous function $g$ on all of $mathbb{R}$, and we have $f=g$ on $S$.



If $S$ contains a negative irrational number $x$, then $g(x) = f(x) = -1$. Let $y$ be any positive number in $S$. If $y$ is rational, we have $g(y) = f(y) = 0$. Then by the continuity of $g$, there would have to be some $z in S$ with $f(z) = g(z) in (-3/4, -1/4)$ which is impossible. If $y$ is irrational, we get a similar contradiction since $g(y) = f(y) = -2$. So $S$ does not contain any negative irrational. Similarly, $S$ does not contain any positive irrational.



So we must have $S subset mathbb{Q}$. But this is similarly impossible. The rationals in $S$ cannot all have the same denominator (in lowest terms), so let $x_1 = p_1/q_1, x_2 = p_2/q_2 in S$, where $q_1 < q_2$. Then by the continuity of $g$ there must be some $y in S$ with $f(y) = g(y) in (frac{1}{q_1}, frac{1}{q_1+1})$, but $f$ never takes on any such value.







share|cite|improve this answer












share|cite|improve this answer



share|cite|improve this answer










answered 6 hours ago









Nate Eldredge

19.6k365109




19.6k365109












  • Slight simplification of the proof: $S$ cannot contain a single rational number, since $f$ is discontinuous at every rational, hence so is $f|_S$ for any dense $S$. Thus $Ssubseteqmathbb Rsetminusmathbb Q$, but then we have problems around $0$.
    – Wojowu
    5 hours ago












  • @Wojowu: It's not quite that simple, I think. For instance, the function $f=1_mathbb{Q}$ is discontinuous at every rational (and every irrational), but its restriction to $S=mathbb{Q}$ is uniformly continuous. So the fact that $f$ is discontinuous at the rationals doesn't immediately rule out the possibility for $S$ to contain rationals. We would have to work a little harder. I guess the key is that $f$ has a "jump" discontinuity at every rational.
    – Nate Eldredge
    5 hours ago












  • You are right, I was a little too quick there. We can either use jump discontinuity as you mention, or we can observe $(q,f(q))$ is an isolated point of the graph.
    – Wojowu
    5 hours ago


















  • Slight simplification of the proof: $S$ cannot contain a single rational number, since $f$ is discontinuous at every rational, hence so is $f|_S$ for any dense $S$. Thus $Ssubseteqmathbb Rsetminusmathbb Q$, but then we have problems around $0$.
    – Wojowu
    5 hours ago












  • @Wojowu: It's not quite that simple, I think. For instance, the function $f=1_mathbb{Q}$ is discontinuous at every rational (and every irrational), but its restriction to $S=mathbb{Q}$ is uniformly continuous. So the fact that $f$ is discontinuous at the rationals doesn't immediately rule out the possibility for $S$ to contain rationals. We would have to work a little harder. I guess the key is that $f$ has a "jump" discontinuity at every rational.
    – Nate Eldredge
    5 hours ago












  • You are right, I was a little too quick there. We can either use jump discontinuity as you mention, or we can observe $(q,f(q))$ is an isolated point of the graph.
    – Wojowu
    5 hours ago
















Slight simplification of the proof: $S$ cannot contain a single rational number, since $f$ is discontinuous at every rational, hence so is $f|_S$ for any dense $S$. Thus $Ssubseteqmathbb Rsetminusmathbb Q$, but then we have problems around $0$.
– Wojowu
5 hours ago






Slight simplification of the proof: $S$ cannot contain a single rational number, since $f$ is discontinuous at every rational, hence so is $f|_S$ for any dense $S$. Thus $Ssubseteqmathbb Rsetminusmathbb Q$, but then we have problems around $0$.
– Wojowu
5 hours ago














@Wojowu: It's not quite that simple, I think. For instance, the function $f=1_mathbb{Q}$ is discontinuous at every rational (and every irrational), but its restriction to $S=mathbb{Q}$ is uniformly continuous. So the fact that $f$ is discontinuous at the rationals doesn't immediately rule out the possibility for $S$ to contain rationals. We would have to work a little harder. I guess the key is that $f$ has a "jump" discontinuity at every rational.
– Nate Eldredge
5 hours ago






@Wojowu: It's not quite that simple, I think. For instance, the function $f=1_mathbb{Q}$ is discontinuous at every rational (and every irrational), but its restriction to $S=mathbb{Q}$ is uniformly continuous. So the fact that $f$ is discontinuous at the rationals doesn't immediately rule out the possibility for $S$ to contain rationals. We would have to work a little harder. I guess the key is that $f$ has a "jump" discontinuity at every rational.
– Nate Eldredge
5 hours ago














You are right, I was a little too quick there. We can either use jump discontinuity as you mention, or we can observe $(q,f(q))$ is an isolated point of the graph.
– Wojowu
5 hours ago




You are right, I was a little too quick there. We can either use jump discontinuity as you mention, or we can observe $(q,f(q))$ is an isolated point of the graph.
– Wojowu
5 hours ago


















draft saved

draft discarded




















































Thanks for contributing an answer to MathOverflow!


  • Please be sure to answer the question. Provide details and share your research!

But avoid



  • Asking for help, clarification, or responding to other answers.

  • Making statements based on opinion; back them up with references or personal experience.


Use MathJax to format equations. MathJax reference.


To learn more, see our tips on writing great answers.





Some of your past answers have not been well-received, and you're in danger of being blocked from answering.


Please pay close attention to the following guidance:


  • Please be sure to answer the question. Provide details and share your research!

But avoid



  • Asking for help, clarification, or responding to other answers.

  • Making statements based on opinion; back them up with references or personal experience.


To learn more, see our tips on writing great answers.




draft saved


draft discarded














StackExchange.ready(
function () {
StackExchange.openid.initPostLogin('.new-post-login', 'https%3a%2f%2fmathoverflow.net%2fquestions%2f319525%2fa-problem-in-real-analysis-of-a-topological-nature%23new-answer', 'question_page');
}
);

Post as a guest















Required, but never shown





















































Required, but never shown














Required, but never shown












Required, but never shown







Required, but never shown

































Required, but never shown














Required, but never shown












Required, but never shown







Required, but never shown







Popular posts from this blog

Trompette piccolo

Slow SSRS Report in dynamic grouping and multiple parameters

Simon Yates (cyclisme)